Blaine works for a battery manufacturing company. he wants to develop a method to test the batteries made each day to determine if they work. which method would provide the most valid results?

Answers

Answer 1

Random sampling 50 batteries throughout the day and testing to see if they would provide most valid results.

A sample is described as a smaller, more manageable representation of a larger group. A smaller population with characteristics of a larger one. A sample is used in statistical analysis when the population size is too large to include all participants or observations in the test.

The sample may be biased if it is taken from the first 25 batteries made each day or the last 25 batteries made each day.

The first and last battery made each day could not be an accurate representation of the total quality of the batteries produced that day if the battery manufacturing process is not consistent thought the day.

Although the battery quality can change during the day, sampling the first 50 batteries produced each day may also add bias into the sample.

The ideal course of action is to randomly select 50 batteries throughout the day, since this helps to ensure that the sample is reflective of the general calibre of batteries manufactured that day.

A more accurate image of the quality of the batteries manufactured can be obtained using this method, which is also more likely to capture any variations in battery quality over the day.

To know more about Random sampling go through:-

https://brainly.com/question/13219833

#SPJ4


Related Questions

Eddie est discutiendo con Tana sobre las probabilidades de los distintos resultados al lanzar tres monedas. Decide lanzar una moneda de un centavo, una de cinco centavos y una de die centavos. ¿ Cuál es la probabilidad de que las tres monedas salgan cruz?

Answers

The probability of getting tails in the three coins would be 0.125 or 12.5%.

How to calculate the probability?

To calculate the probability of an event happening, first, we need to identify the rate of the desired outcome versus the total possible outcomes. Moreover, to determine the total probability of two or more events happening we need to calculate the probability of each event and then multiply the results.

Probability of getting tails in any of the three coins:

1 / 2 = 0.5

Total probabilityy:

0.5 x 0.5 x 0.5 = 0.125 or 12.5%

Learn more about probability in https://brainly.com/question/30034780

#SPJ1

Theorem: "If a and m are relatively prime integers and m > 1, then an inverse of a modulo m exists. Furthermore, this inverse is unique modulo m. (That is, there is a unique positive integer a less than m that is an inverse of a modulo m and every other inverse of a modulo m is congruent to a modulo m.)"Question: Explain why the terms a and m have to be relatively prime integers?

Answers

The reason why the terms a and m have to be relatively prime integers is that it is the only way to make sure that ax≡1 (mod m) is solvable for x within the integers modulo m.

Theorem:"If a and m are relatively prime integers and m > 1, then an inverse of a modulo m exists. Furthermore, this inverse is unique modulo m. (That is, there is a unique positive integer a less than m that is an inverse of a modulo m and every other inverse of a modulo m is congruent to a modulo m.)"If a and m are relatively prime integers and m > 1, then an inverse of a modulo m exists. Furthermore, this inverse is unique modulo m. (That is, there is a unique positive integer a less than m that is an inverse of a modulo m and every other inverse of a modulo m is congruent to a modulo m.)The inverse of a modulo m is another integer, x, such that ax≡1 (mod m).

This theorem has an interesting explanation: if a and m are not co-prime, then there is no guarantee that ax≡1 (mod m) has a solution in Zm. The reason for this is that if a and m have a common factor, then m “absorbs” some of the factors of a. When this happens, we lose information about the congruence class of a, and so it becomes harder (if not impossible) to undo the multiplication by .This is the reason why the terms a and m have to be relatively prime integers.

To know more about function click here :

https://brainly.com/question/12976257

#SPJ11

PLEASE HELP!!! WILL MARK BRANLIEST!!!

Answers

Answer:

The point z = 3+4i is plotted as a blue dot, and the two square roots are plotted as a red dot and a green dot. The magnitudes of z and its square roots are shown by the radii of the circles centered at the origin.

Step-by-step explanation:

qrt(z) = +/- sqrt(r) * [cos(theta/2) + i sin(theta/2)]

where r = |z| = magnitude of z and theta = arg(z) = argument of z.

Calculate the magnitude of z:

|r| = sqrt((3)^2 + (4)^2) = 5

And the argument of z:

theta = arctan(4/3) = 0.93 radians

Now, find the two square roots of z:

sqrt(z) = +/- sqrt(5) * [cos(0.93/2) + i sin(0.93/2)]

= +/- 1.58 * [cos(0.47) + i sin(0.47)]

= +/- 1.58 * [0.89 + i*0.46]

Using a calculator, simplify this expression to:

sqrt(z) = +/- 1.41 + i1.41 or +/- 0.2 + i2.8

in fig. 8-25, a block slides along a track that descends through distance h.the track is frictionless except for the lower section. there the block slides to a stop in a certain distance d because of friction. (a) if we decrease h,will the block now slide to a stop in a distance that is greater than, less than, or equal to d? (b) if, instead, we increase the mass of the block, will the stopping distance now be greater than, less than, or equal to d?

Answers

a block slides along a track that descends through distance h. The track is frictionless except for the lower section. There the block slides to a stop in a certain distance d because of friction. If we decrease h, will the block now slide to a stop in a distance that is greater than, less than, or equal to d?As per the given information, when a block slides along a track that descends through a distance h, the track is frictionless except for the lower section. There the block slides to a stop in a certain distance d because of friction. Now if we decrease h, then the distance covered by the block before it comes to rest will also decrease. So the block will slide to a stop in a distance that is less than d. Hence the answer is less than d.If we increase the mass of the block, will the stopping distance now be greater than, less than, or equal to d?

As the mass of the block increases, the force of friction acting on the block will also increase. Hence the stopping distance will also increase. So the stopping distance now will be greater than d. Hence the answer is greater than d.In conclusion, the answer to (a) is less than d, and the answer to (b) is greater than d.

for such more questions on conclusion

https://brainly.com/question/26093731

#SPJ11

A sphere is to be designed with a radius of 72 in. Use differentials to estimate the maximum error when measuring the volume of the sphere if the possible error in measuring the radius is 0.5 in. 4 (Hint: The formula for the volume of a sphere is V(r) = ²³.) O 452.39 in ³ O 16,286.02 in ³ O 65,144.07 in ³ O 32,572.03 in ³

Answers

By using differentials to estimate the maximum error when measuring the volume of the sphere if the possible error in measuring the radius is 0.5. It will be 32,572.03 in³. Which is option (d).

How to measure the maximum error while measuring the volume of a sphere?

The possible error in measuring the radius of the sphere is 0.5 in

The formula for the volume of a sphere is given by V(r) = 4/3πr³

The volume of the sphere when r=72 in is given by V(72) = 4/3π(72)³

When r= 72 + 0.5 in= 72.5 in, the volume of the sphere can be calculated using the formula:

V(72.5) = 4/3π(72.5)³

The difference between these two volumes, V(72) and V(72.5), gives us the maximum error while measuring the volume of a sphere. It can be calculated as follows:

V(72.5) - V(72) = 4/3π(72.5)³ - 4/3π(72)³= 4/3π [ (72.5)³ - (72)³ ]= 4/3π [ (72 + 0.5)³ - 72³ ]= (4/3)π [ 3(72²)(0.5) + 3(72)(0.5²) + 0.5³ ]≈ (4/3)π [ 777.5 ]= 3.28 × 10⁴ in³

Therefore, the maximum error while measuring the volume of a sphere with a radius of 72 in, where the possible error in measuring the radius is 0.5 in, is approximately 3.28 × 10⁴ in³ or 32,572.03 in³. Therefore coorect option is (D).

To know more about the maximum error: https://brainly.com/question/13370015

#SPJ11

Using c use the best term to identify the following.

Answers

The correct definition for the lines drawn to circle with centre C are:

FA is an secant.CD is the radius of the circle.DE is the diameter.EB is the tangent on the circle.Explain about the circle?

A circle is a spherical shape without boundaries or edges.

A radius describes the distance radiating from the centre.The Diameter passes through the centre of the circle in a straight line.The distance travelled through a circle is its circumference.A line that precisely crosses a circle at one point is said to be tangent.The circular region is divided into two sections by a circle's chord. The term "circular segment" refers to each component.The major segment and minor segment are distinguished by the arcs they contain. The major segment contains the minor arc.

Thus, on the basis of propertied of circle, the correct definition for the lines drawn to circle with centre C are:

FA is an secantCD is the radius of the circle.DE is the diameter.EB is the tangent on the circle.

know more about the circle,

https://brainly.com/question/24375372

#SPJ1

In data analytics, a _____ refers to all possible data values in a certain dataset

Answers

In data analytics, a population refers to all possible data values in a certain dataset.

What is data analytics?

Data analytics is a set of procedures and processes for examining datasets in order to draw conclusions from the information they contain, often aided by specialized systems and software. Organizations use data analytics to aid decision-making, increase efficiency, and evaluate outcomes.

The population and sample are two concepts in statistics. The population and sample are two concepts in statistics. The population is the entire set of objects or individuals being studied, while the sample is a subset of the population that is chosen for analysis. The sample is a subset of the population, chosen at random or according to some other criteria in order to represent the population as a whole.

Learn more about dataset here: https://brainly.com/question/30034464

#SPJ11

Triangle ABC has coordinates A(4,1), B(5,9),and C (2,7). If the triangle is translated 7 units to left, what are the coordinates of B'? ​

Answers

Answer:

(-2,9)

Step-by-step explanation:

when moving it 5 units left on the x axis it would be 5-7

So in turn you would be given (-2,9)

Because the y stays the same you would still have (?,9)

Can anyone solve this ???

Answers

The result (recurrent value), A = sum j=1 to 89 ln(j), is true for every n. This is the desired result.

How do you depict a relationship of recurrence?

As in T(n) = T(n/2) + n, T(0) = T(1) = 1, a recurrence or recurrence relation specifies an infinite sequence by explaining how to calculate the nth element of the sequence given the values of smaller members.

We can start by proving the base case in order to demonstrate the first portion through recurrence. Let n = 1. Next, we have:

Being true, ln(a1) = ln(a1). If n = k, let's suppose the formula is accurate:

Sum j=1 to k ln = ln(prod j=1 to k aj) (aj)

Prod j=1 to k aj * ak+1 = ln(prod j=1 to k+1 aj)

(Using the logarithmic scale) = ln(prod j=1 to k aj) + ln(ak+1)

Using the inductive hypothesis, the property ln(ab) = ln(a) + ln(b)) = sum j=1 to k ln(aj) + ln(ak+1) = sum j=1 to k+1 ln (aj)

(b), we can use the just-proven formula:

A = ln(1, 2,...) + ln + ln (89)

= ln(j=1 to 89) prod

sum j=1 to 89 ln = ln(prod j=1 to 89 j) (j).

To know more about hypothesis visit:-

https://brainly.com/question/29519577

#SPJ1

a) if lisa's score was 83 and that score was the 29th score from the top in a class of 240 scores, what is lisa's percentile rank? (round your answer to the nearest whole number.)

Answers

Lisa's percentile rank is approximately 88%.

Percentile rank is a statistical measure that indicates the percentage of scores that fall below a particular score in a given distribution of data. It is commonly used to describe the relative position of a particular score in a set of scores.

If Lisa's score was 83 and that score was the 29th score from the top in a class of 240 scores, then her percentile rank can be calculated using the following formula:

Percentile Rank = [(Number of scores below Lisa's score) ÷ (Total number of scores)] × 100

Percentile Rank = [(240 - 29) ÷ 240] × 100

Percentile Rank = (211 ÷ 240) × 100

Percentile Rank = 0.8792 × 100

Percentile Rank ≈ 88 (rounded to the nearest whole number)

Therefore, her percentile rank is approximately 88%.

Learn more about percentile here: https://brainly.com/question/24245405.

#SPJ11

HELP PLS combine the like terms 3x+5-x+3+4x​

Answers

Answer:

3x, 4x | 5, 3

Step-by-step explanation:

Solve equation for x
216=6^4x+5

Answers

Answer: x=211/1296

Step-by-step explanation:

use the slicing method to find the volume of the solid whose base is the region inside the circle with radius 3 if the cross sections taken parallel to one of the diameters are equilateral triangles.

Answers

The volume of the solid whose base is the region inside the circle with radius 3 if the cross sections taken parallel to one of the diameters are equilateral triangles is 81/2*\sqrt3 by using the slicing method.

To find the volume of the solid whose base is the region inside the circle with radius 3, we need to integrate the area of the cross sections taken parallel to one of the diameters, which are equilateral triangles.

Let's consider a cross section of the solid taken at a distance x from the center of the circle.

Since the cross section is an equilateral triangle, all its sides have the same length.

Let  this length be y. Since the triangle is equilateral, its height can be found using the Pythagorean theorem as follows:

[tex]height = \sqrt{(y^2 - (y/2)^2)} = \sqrt{(3/4y^2)}= \sqrt{3/2y}[/tex]

Therefore, the area of the cross section at a distance x from the center of the circle is:

[tex]A(x) = (1/2)y\sqrt{3/2y} = \sqrt{3/4y^2}[/tex]

Now, we need to integrate this area over the range of x from -3 to 3 (since the circle has radius 3):

[tex]V = \int\ [-3,3]\sqrt{3/4*y^2} dx[/tex]

To find the limits of integration for y, we need to consider the equation of the circle:

[tex]x^2 + y^2= 3^2[/tex]

Solving for y, we get:

[tex]y =\pm\sqrt{(3^2 - x^2)}=\pm\sqrt{(9^2 - x^2)}[/tex]

Since we want the cross sections to be equilateral triangles, we know that y is equal to the height of an equilateral triangle with side length equal to the diameter of the circle, which is 2*3 = 6. Therefore, we can write:

[tex]y = 3*\sqrt{3}[/tex]

Substituting this into the integral, we get:

[tex]V = \int\ [-3,3] \sqrt{3/4*(3\sqrt3)^2} dx[/tex]

[tex]= \int\ [-3,3] 27/4*\sqrt{3} dx[/tex]

Integrating, we get:

[tex]V = [27/4\sqrt{3x}]*[-3,3][/tex]

[tex]= 81/2*\sqrt{3}[/tex]

Therefore, the volume of the solid is [tex]81/2*\sqrt3[/tex]cubic units

To practice more questions about 'volume of solid':

https://brainly.com/question/20284914

#SPJ11

pls help Are the following lines parallel, perpendicular, or neither?

y = 2/3x − 4

y = −3/2x − 7

Responses


Parallel

Perpendicular

Neither

Answers

Answer:

Perpendicular.

Step-by-step explanation:

To determine whether the two lines are parallel, perpendicular, or neither, we need to compare their slopes.

The slope-intercept form of a line is y = mx + b, where m is the slope and b is the y-intercept. So we can rewrite the given equations in this form

y = 2/3x - 4 ==> slope = 2/3

y = -3/2x - 7 ==> slope = -3/2

Two lines are parallel if and only if their slopes are equal. Therefore, since the slopes of the two lines are different (2/3 and -3/2), they cannot be parallel.

Two lines are perpendicular if and only if their slopes are negative reciprocals of each other. That is, if the product of their slopes is -1. Therefore, we can check if the product of the slopes of the two lines is -1

(2/3) * (-3/2) = -1

Since the product of the slopes is -1, the two lines are perpendicular.

Therefore, the answer is: perpendicular.

A hawk flying at 19 m/s at an altitude of 228 m accidentally drops its prey. The parabolic trajectory of the falling prey is described by the equation y = 228 − x^2/57 until it hits the ground, where y is its height above the ground and x is its horizontal distance traveled in meters. Calculate the distance traveled by the prey from the time it is dropped until the time it hits the ground. Express your answer correct to the nearest tenth of a meter.

Answers

The parabolic trajectory of the falling prey can be described by the equation y = 228 – x2/57, where y is the height above the ground and x is the horizontal distance traveled in meters. In this case, the prey was dropped at a height of 228 m and flying at 19 m/s. To calculate the total distance traveled by the prey, we can use the equation for the parabola to solve for x.

We can rearrange the equation y = 228 – x2/57 to solve for x, which gives us[tex]x = √(57*(228 – y))[/tex]. When the prey hits the ground, the height (y) is 0. Plugging this into the equation for x, we can calculate that the total distance traveled by the prey is[tex]x = √(57*(228 - 0)) = √(57*228) = 84.9 m.\\[/tex] Expressing this answer to the nearest tenth of a meter gives us the final answer of 84.9 m.

for such more questions on  parabolic trajectory

https://brainly.com/question/13244761

#SPJ11

Sophie invested $92,000 in an account paying an interest rate of 6 1/8% compounded

continuously. Damian invested $92,000 in an account paying an interest rate of 6 5/8%

compounded monthly. After 14 years, how much more money would Damian have in

his account than Sophie, to the nearest dollar?

Answers

Answer:

Step-by-step explanation:

To solve this problem, we need to use the formula for compound interest:

A = P*e^(rt)

where A is the final amount, P is the principal (initial investment), e is the base of the natural logarithm (approximately 2.71828), r is the interest rate (expressed as a decimal), and t is the time (in years).

For Sophie's account, we have:

P = $92,000

r = 6 1/8% = 0.06125 (as a decimal)

t = 14 years

A = 92000*e^(0.06125*14)

A = $219,499.70 (rounded to the nearest cent)

For Damian's account, we have:

P = $92,000

r = 6 5/8% = 0.06625/12 = 0.005521 (as a monthly decimal rate)

t = 14*12 = 168 months

A = 92000*(1+0.005521)^168

A = $288,947.46 (rounded to the nearest cent)

Now we can subtract Sophie's final amount from Damian's final amount to find the difference:

Difference = $288,947.46 - $219,499.70

Difference = $69,447.76

Therefore, Damian would have about $69,448 more in his account than Sophie, to the nearest dollar.

The equation and graph show the distance traveled by a covertible and a limousine in miles, y, as a function of time in hours, x.

Answers

The rate of change of the distance for limousine is less than the rate of change of the convertible.

What is rate of change?

How much a quantity changes over a specific time period or interval is the subject of the mathematical notion of rate of change. Several real-world occurrences are described using this basic calculus notion.

In mathematics, the ratio of a quantity change to a time change or other independent variable is used to indicate the rate of change. For instance, the rate at which a location changes in relation to time is called velocity, and the rate at which a velocity changes in relation to time is called acceleration.

The equation of the distance travelled by the convertible is given as:

y = 35x

The equation of the limousine can be calculated using the coordinates of the graph (1, 30) and (2, 60).

The slope is given as:

slope = (change in y) / (change in x) = (60 - 30) / (2 - 1) = 30

Using the point slope form:

y - 30 = 30(x - 1)

y = 30x

So the equation of the limousine is y = 30x.

Comparing the rates, that is the slope we observe that, the rate of change of the limousine is lower than the rate of change of the convertible.

Hence, the rate of change of the limousine is less than the rate of change of the convertible.

Learn more about rate of change here:

https://brainly.com/question/29181502

#SPJ1

Customer five had a $5.00 off coupon, but still has to pay the 4.5% sales tax. How much do they end up paying?

Answers

Sure, I can help you with this. To calculate the amount that Customer five will end up paying with their $5.00 off coupon and 4.5% sales tax, we will use the following formula: final amount = original amount - coupon - (original amount * tax rate).

In this case, the original amount is $5.00, the coupon is $5.00, and the tax rate is 4.5%. Plugging these values into the formula, we get:

final amount = 5.00 - 5.00 - (5.00 * 0.045)

final amount = 5.00 - 5.00 - 0.225

final amount = 4.775

Therefore, Customer five will end up paying $4.775 after their coupon and the sales tax.

If all other factors are held constant, which of the following results in an increase in the probability of a Type II error? a. The true parameter is farther from the value of the null hypothesis. b. The sample size is increased. c. The significance level is decreased d. The standard error is decreased. e. The probability of a Type II error cannot be increased, only decreased

Answers

If all other factors are held constant, then the true parameter is farther from the value of the null hypothesis which is an increase in the probability of a Type II error.The correct option is A.

The true parameter is farther from the value of the null hypothesis.

When the true parameter is farther away from the value of the null hypothesis, it increases the probability of a Type II error. This is because the null hypothesis will have a harder time rejecting the true parameter.

The other factors - increasing sample size, decreasing significance level, and decreasing standard error - all result in a decreased probability of a Type II error.

To learn more about the null hypothesis:

https://brainly.com/question/15980493

#SPJ11

Each of these measures is rounded to nearest whole: a=5cm and b=3cm Calculate the upper bound of a +b

Answers

The upper bound of a + b can be found by adding the upper bounds of a and b.

For a = 5cm, the nearest whole number is 5. The upper bound would be the midpoint between 5 and 6, which is 5.5.

For b = 3cm, the nearest whole number is 3. The upper bound would be the midpoint between 3 and 4, which is 3.5.

So the upper bound of a + b is:

5.5 + 3.5 = 9

Therefore, the upper bound of a + b is 9cm.

there are 20 rows of seats on a concert hall: 25 seats are in the 1st row, 27 seats on the 2nd row, 29 seats on the 3 rd row, and so on. if the price per ticket is $32, how much will be the total sales for a one-night concert if all seats are taken?

Answers

Answer:

Step-by-step explanation:

To solve this problem, we need to find out how many seats there are in total, and then multiply that by the price per ticket.

To find the total number of seats, we need to add up the number of seats in each row. We can use the formula for an arithmetic sequence to do this:

S = n/2 * (a + l)

where S is the sum of the sequence, n is the number of terms, a is the first term, and l is the last term.

In this case, we have:

n = 20 (since there are 20 rows)

a = 25 (since there are 25 seats in the first row)

d = 2 (since the difference between each row is 2 seats, the common difference is 2)

We can use d to find the last term as well:

l = a + (n-1)*d

l = 25 + (20-1)*2

l = 25 + 38

l = 63

Now we can plug these values into the formula:

S = 20/2 * (25 + 63)

S = 10 * 88

S = 880

So there are 880 seats in total.

To find the total sales, we just need to multiply by the price per ticket:

total sales = 880 * $32

total sales = $28,160

Therefore, the total sales for a one-night concert with all seats taken would be $28,160.

Can anyone solve this problem please? Thanks!

Answers

The trapezoid has a surface area of 480 square units.

What is the measurement for a trapezoid's area?

So, a trapezoid measured in feet offers an area in square feet; one measured in millimetres gives an area in square centimetres; and so on. If it's simpler for you, you can add the lengths of the bases and then divide the total by two. Keep in mind that multiplication by 12 is equivalent to dividing by 2.

We must apply the formula for a trapezoid's area to this issue in order to find a solution:

[tex]A = (1/2) * (a + b) * h[/tex]

where h is the trapezoid's height (or altitude) and a and b are the lengths of its parallel sides.

The values for a, b, and h are provided to us, allowing us to change them in the formula:

A = (1/2) * (20 + 60) * 12

A = (1/2) * 80 * 12

A = 480 square units

To know more about trapezoid visit:-
https://brainly.com/question/8643562

#SPJ1

Which equation is equivalent to pq=r?
Responses

A) p=logR q
B) p=logQ r
C) q=logR p
D) q=logP r

Answers

The equation is equivalent to pq=r is option (C) q=logR p

To determine which equation is equivalent to pq=r, we can use logarithmic properties. Taking the logarithm of both sides of the equation, we get

log(pq) = log(r)

Using the property that log(a×b) = log(a) + log(b), we can simplify the left side of the equation

log(p) + log(q) = log(r)

Now, we can compare this expression to each of the answer choices

A) p = logR q

Substituting this into the equation, we get

log(p) + logR(q) = log(r)

This is not equivalent to our expression, so A is not the correct answer.

B) p = logQ r

Substituting this into the equation, we get

log(logQ r) + log(q) = log(r)

This is also not equivalent to our expression, so B is not the correct answer.

C) q = logR p

Substituting this into the equation, we get

log(p) + logR(q) = log(r)

This matches our expression, so C is the correct answer.

D) q = logP r

Substituting this into the equation, we get

log(p) + log(q) = log(logP r)

This is not equivalent to our expression, so D is not the correct answer.

Therefore, the correct option is (C)  q=logR p

Learn more about logarithmic properties here

brainly.com/question/30226560

#SPJ4

An electric dipole with its center located at the origin of a Cartesian coordinate system oscillates along the z axis, creating an electromagnetic wave. At a position on the y axis far from the origin, what is the polarization of the wave and which axis are the magnetic (a) The wave is polarized parallel to the a axis and the magnetic field lines are parallel to b The wave is polarized parallel to the z axis and the magnetic field lines are parallel to (c) The wave is polarized parallel to the y axis and the magnetic field lines are parallel to (d) The wave is polarized parallel to the y axis and the magnetic field lines are parallel to (e) The wave is polarized parallel to the z axis and the magnetic field lines are parallel to field lines parallel to? the y axis the axis the r axis the z axis the z axis

Answers

The wave is polarized parallel to the y-axis, and the magnetic field lines are parallel to the x-axis. Here option D is the correct answer.

The oscillating electric dipole along the z-axis creates an electromagnetic wave with electric and magnetic fields perpendicular to each other and to the direction of wave propagation. At a position on the y-axis far from the origin, the electric field will be parallel to the y-axis.

The polarization of the wave refers to the orientation of the electric field vector. Since the electric field is parallel to the y-axis, the wave is polarized parallel to the y-axis.

According to the right-hand rule, the direction of the magnetic field lines will be perpendicular to both the electric field and the direction of wave propagation, which is along the z-axis. Therefore, the magnetic field lines will be parallel to the x-axis.

To learn more about magnetic fields

https://brainly.com/question/11514007

#SPJ4

I need some help with this​

Answers

Answer:

12

Step-by-step explanation:

i think its right

If G is a group with subgroups A, B of orders m, n, respectively, where m and n are relatively prime, prove that the subset of G, AB = {abla E Ab E B}, has mn distinct elements.

Answers

The number of distinct elements of AB = m n.

Given that G is a group with subgroups A and B of orders m and n, respectively, where m and n are prime, we need to prove that the subset of G, AB = {abla E Ab E B}, has m n distinct elements. Step-by-step. Let, G is a group with subgroups A and B of orders m and n, respectively. Since, m and n are relatively prime, then we have gcd(m, n) = 1.By Lagrange's Theorem, the order of any subgroup of G divides the order of G.

Hence, the order of G is equal to the product of the orders of A and B, i.e. |G| = |A| * |B| = m * n Let, a and a' be two distinct elements of A and b and b' be two distinct elements of B. Thus, a and a' generate distinct subgroups of G, i.e.  ≠  and b and b' generate distinct subgroups of G, i.e.  ≠ .Now, the number of distinct elements of AB = {abla E Ab E B} is equal to |A||B| since any two elements ab and a'b' of AB will be distinct if either a and a' are distinct or b and b' are distinct or both are distinct. Hence, the number of distinct elements of AB = m n.

Learn more about Distinct

brainly.com/question/20739236

#SPJ11

kernel composition rules 2 1 point possible (graded) let and be two vectors of the same dimension. use the the definition of kernels and the kernel composition rules from the video above to decide which of the following are kernels. (note that you can use feature vectors that are not polynomial.) (choose all those apply. )
a. 1
b. x.x’
c. 1+ x.x’
d. (1+ x.x’)^2
e. exp (x+x’), for x.x’ ER
f. min (x.x’) for x.x’ E Z

Answers

Answer:

Step-by-step explanation:

kernel composition rules 2 1 point possible (graded) let and be two vectors of the same dimension. use the the definition of kernels and the kernel composition rules from the video above to decide which of the following are kernels. (note that you can use feature vectors that are not polynomial.) (choose all those apply. )

Please urgent need the work and answer
X=3.2
Y=6.1
Z=0.2

XZ +Y2

Answers

Answer: 12.84

Step-by-step explanation:

if x = 3.2 and y = 6.1  and Z = 0.2

then plug in the numbers

(3.2)(0.2) + (6.1)(2)

0.64 + 12.2 = 12.84

Any variable next to a number means multiplication.

if I was wrong lmk

#1 Brainlist!
Answer and show steps and I will make you brainlist.

Answers

Answer:

Multiplying the second equation by 5, we get:

15x + 20y = 180

Now, we can add this equation to the first equation:

26x = 208

x = 8

Substituting x = 8 in the second equation:

3(8) + 4y = 36

4y = 12

y = 3

Therefore, the solution to the system is (8, 3).

The island of Martinique has received $32,000
for hurricane relief efforts. The island’s goal is to
fundraise at least y dollars for aid by the end of
the month. They receive donations of $4500
each day. Write an inequality that represents this
situation, where x is the number of days.

Answers

An inequality representing the amount that the island of Martinique can received for hurricane relief efforts, where x is the number of days is y ≤ 32,000 + 4,500x.

What is inequality?

Inequality is an algebraic statement that two or more mathematical expressions are unequal.

Inequalities can be represented as:

Greater than (>)Less than (<)Greater than or equal to (≥)Less than or equal to (≤)Not equal to (≠).

The total amount received by the island = $32,000

The daily receipt of donations = $4,500

Let the number of days = x

Let the funds raised for aid = y

Inequality:

y ≤ 32,000 + 4,500x

Thus, the inequality for the funds that the island can fundraise for hurricane relief aid by the end of the month is y ≤ 32,000 + 4,500x.

Learn more about inequalities at https://brainly.com/question/24372553.

#SPJ1

Other Questions
Which evidence suggests that some of the first life on earth was prokaryotic bacteria? an intense fear and avoidance of negative public scrutiny, public embarrassment, humiliation, or social interaction is called by? in a peptide bond, which parts of the two amino acids are joined together? an organization uses a database management system (dbms) as a repository of data. the dbms in turn supports a number of end-user-developed applications. some of the applications update the database. in evaluating the control procedures over access and use of the database, the auditor will be most concerned that Lara opened a savings account 1 year ago. The account earns 11% interest, compoundedcontinuously. If the current balance is $7,000.00, how much did she deposit initially?Round your answer to the nearest cent. Unless a reader is rhetorically savvy, he/she may get so lost in the rhetorical effectiveness of the message (the "bells" and "whistles") that it becomes easy to lose awareness that they are viewing the subject matter through a limited lens AND that they are being manipulated to think in a particular way about the subject. Question 1 options: True False Solve please geometry, solve for x a fixed amount of a molecular substance in the liquid phase is placed in a flask at constant temperature. the flask is closed and is allowed to come to equilibrium. select all the statements that correctly describe the processes occurring in the flask. multiple select question. a. the relative amounts of liquid and vapor in the flask remain constant. b. molecules are leaving and entering the liquid phase at the same rate. c. no changes are occurring because the system is at equilibrium. d. the amount of liquid remains the same because evaporation is no longer occurring. a fraction nonconforming control chart is to be established with a center line of 0.01 and two-sigma control limits. (a) how large should the sample size be if the lower control limit is to be nonzero? (b) how large should the sample size be if we wish the probability of detecting a shift to 0.04 to be 0.50? Read lines 2228 from Act IV, Scene 1 of the play:Paris. Come you to make confession to this father?Juliet. To answer that, I should confess to you.Paris. Do not deny to him that you love me.Juliet. I will confess to you that I love him.Paris. So will ye, I am sure, that you love me.Juliet. If I do so, it will be of more price,Being spoke behind your back, than to your face.Select two ways in which the dramatic irony of these lines contribute to the tension that is building in the play.ResponsesA,Paris is depending on enemies for guidance.b,Paris is depending on enemies for guidance.c,Paris is inevitably going to face harsh truths.d,Paris is inevitably going to face harsh truths.e.Juliet is actively trying to ruin Pariss life.f,Juliet is actively trying to ruin Pariss life.g,Paris is totally unaware of Juliets true feelings.h,Paris is totally unaware of Juliets true feelings.i,Juliet is acting anxiously out of the ordinary. one electron collides elastically with a second electron initially at rest. after the collision, the radii of their trajectories are 0.00 cm and 3.00 cm. the trajectories are perpendicular to a uniform magnetic field of magnitude 0.0350 t. determine the energy (in kev) of the incident electron. Jenny took the car, the bus, and the train to get home in time.What form of punctuation is missing?O A. No punctuation is missing.OB.A periodOC.A commaOD. A semicolonLast three times I have tried to take a picture of my question. Nothing comes up that resembles any of it. I dont know whats wrong with this app but its not helping. I NEED HELP ASAP. How do the ideas of the talent search finalist support the main idea of the text these. Teens have some ideas for stopping climate change commonlit 13. The Federal Deposit Insurance Corporation is an agency created by the U.S. Congress tomonitor and assist financial private institutions to protect consumers. This agency fulfills whichfunction of government?A. provide economic assistanceB. provide economic securityC. provide leadershipD. provide national security 314 VISION SACCO su se CRANE SACCO earth as required $2,000 per group metic of 12% q 1% of the ban ng fee f LX The UMOJA SACCO we frase fee of sk: C p needs Cx 10 milion to start up a maize growing project. As a Senior advise this group on how to choose the best SACCO chapter, you have: nt how to apply business authmetic by calculating profit loss, commiss rest Insu which of the following type of regulatory authority issues rules that have the impact of laws? question 58 options: quasi-legislative in personam quasi-executive quasi-regulatory quasi in rem The Jones family has two dogs whose ages add up to 15 and multiply to 44. How old is each dog? Today, organizations are using personal computers for data presentation because personal computer use compared to mainframe use is morea. Controllable.b. Cost effective.c. Reliable.d. Conducive to data integrity. Los venecianos exportaron _______________ a los otomanos. Generalmente se tejan en otras ciudades como ___________ queEneran tan valiosos porque su color permaneca vibrante. Esto se debe a que fueron teidos con un qumico llamado alumbre, que se encontraba principalmente en Anatolia, ubicado en ___________________ ___________________. Ayuda por favor es para hoy which of the following pharmacologic features is shared by all of the medications that have received fda approval for maintenance treatment of opioid use disorder?A. FDA approved medications for maintenance or detoxificationB. Medications in schedules III, IV, or VC. Buprenorphine or Buprenorphine/naloxoneD. Methadone